LSAT and Law School Admissions Forum

Get expert LSAT preparation and law school admissions advice from PowerScore Test Preparation.

 Administrator
PowerScore Staff
  • PowerScore Staff
  • Posts: 8917
  • Joined: Feb 02, 2011
|
#34894
Complete Question Explanation

Assumption. The correct answer choice is (E)

Acme Corporation has decided to move its company from Milltown to Ocean View, despite the fact
that most people who work for Acme cannot afford the housing available within a half hour of the
new location in Ocean View. The author of the stimulus concludes that after the company moves,
most of its workers can look forward to longer than a 30-minute commute. The components of the
argument break down as follows:

..... Premise: ..... Acme is moving from Milltown to Ocean View.

..... Premise: ..... Most Acme employees cannot afford to live within 30 minutes of the new
..... ..... ..... ..... location.

..... Conclusion: ..... After the move, most Acme employees will have longer than a 30-minute
..... ..... ..... ..... commute.

The question that follows is an Assumption question, so the correct answer choice will provide an
assumption that the argument above relies upon; the Assumption Negation Technique can be applied
to confirm the right answer choice.

Answer choice (A): The author of the stimulus does not mention how many Acme employees live
within 30 minutes of the company’s current location in Milltown, so this is not an assumption on
which the author’s argument is based.

Answer choice (B): The author of the stimulus does not mention whether the company chairman has
good reason to move, so this assumption is not necessary to the argument.

Answer choice (C): The author’s argument does not assume that the company chairperson stands
alone in the desire to move the company. To confirm that this assumption is not required by the
argument, we can logically negate and note the effects:

There is at least one other company employee who wants to move the company to Ocean View.

Clearly when we negate this choice, and take away this assumption, it does not weaken the argument
presented in the stimulus, confirming that this is not an assumption required by the author.

Answer choice (D): The current commute time of the majority of Acme employees is not relevant
to the author's argument. Since this is not an assumption on which the stimulus' argument relies, this
choice should be ruled out of contention.

Answer choice (E): This is the correct answer choice.The author’s conclusion relies upon the
assumption that the move will not be accompanied by a significant raise. To confirm that the
author’s argument requires this assumption, we can take it away, or logically negate it, to see
whether this weakens the argument. In this case, the negated version would be as follows:

Acme’s move will be accompanied by a significant pay raise.

If this is the case, that would defeat the author’s conclusion, because it could allow some of the
referenced employees to afford housing closer to the new location.
 mkuo
  • Posts: 24
  • Joined: Nov 06, 2012
|
#11508
Hi,

I got this one right but upon reviewing, it got me thinking a bit more.

What if, even with the significant pay raise for Acme employees, they STILL CAN'T afford to live within 30-min of Ocean View?

If I understand assumption negation correctly, the negation will effectively destroy the argument; negating (E) doesn't necessary nail it down that at least some employees will now be able to afford it. I don't see how the premise "Most Acme employees cannot afford housing within 30-min..." is definitely affected, only that it's possible.

(E) is no doubt a strengthener. But shouldn't a required assumption be something like "Most Acme employees who cannot afford housing within 30-min commute of OV are NOT REPLACED by people who can afford (or people who already live in 30-min of OV)."
Negating that assumption would definitely destroy the conclusion.

Thoughts?
 Steve Stein
PowerScore Staff
  • PowerScore Staff
  • Posts: 1153
  • Joined: Apr 11, 2011
|
#11540
Hi,

The correct answer to assumption questions can be difficult to prephrase with precision, so it can be dangerous to go into the analysis with too much conviction about what the answer will be. Remember, the right answer will be the one presented that provides an assumption required by the argument.

(E) is the right answer because the author’s conclusion relies on the assumption that the move will not be accompanied by a significant raise. To confirm that the author’s argument requires this assumption, we logically negate it, to see whether this weakens the argument. In this case, the negated version would be as follows:

Acme's move will be accompanied by a raise.

If this is the case, that would defeat the author’s conclusion, because it could allow some of the referenced employees to afford housing closer to the new location.

I hope that's helpful--please let me know whether this is clear.

Thanks!

~Steve
 mkuo
  • Posts: 24
  • Joined: Nov 06, 2012
|
#11543
Hi Steve,

Thanks for your explanation! I just read again the section on Assumption Negation... somehow I took it as the negation has to completely destroy the argument. Upon reading it again it appears that it just requires weakening...

I"m glad you straightened that out for me. Thank you.
 jrc3813
  • Posts: 53
  • Joined: Apr 16, 2017
|
#36838
I may have over thought this question but I picked D because the argument was basically saying that the company move would cause most employees to have a commute longer than 30 minutes. So I thought that an assumption would be that employees don't already have a commute over 30 minutes. In order to show that a cause created an effect wouldn't the effect have to be absent first?

I think also E seemed too weak to be the right answer so I was drawn to D. Even if they got a significant pay raise they wouldn't necessarily move closer to the new location. Is it because the conclusion depends on the premise that they can't afford housing that is within 30 minutes? So if they can afford it it means the argument isn't necessarily correct?

It seems more like instead of destroying the argument in the sense of showing that conclusion will not happen, it shows that it doesn't necessarily follow. Is that the right way if looking at it?
 Adam Tyson
PowerScore Staff
  • PowerScore Staff
  • Posts: 5153
  • Joined: Apr 14, 2011
|
#36893
Hey jrc3813, you may be overthinking things a bit when you analyzed answer D on this one. We don't know whether the employees can currently afford housing closer than 30 minutes to work, and we don't know anything about their current commute. All we know is that right now, at their current income levels, they cannot afford to live within 30 minutes of the new location. Our author then concludes that they will end up with a 30+ minute commute. It's not about how their commute will change, but only about what it will be like when they move. Not necessary to hold this up to causal scrutiny, I think.

The negation of answer E suggests that the current income levels will change for the better. If that's so, we can no longer use the current income levels to determine what they will be able to afford once the company moves, and the conclusion has lost all of its support. Sure, they still might not make enough to afford closer housing, but there's no longer any evidence that they won't be able to afford it. As you correctly surmised, when all the support for the conclusion is removed, the conclusion no longer follows. It might be true, but there's no reason any more to believe it.

Good work!
 ShannonOh22
  • Posts: 70
  • Joined: Aug 15, 2019
|
#68231
Adam Tyson wrote:Hey jrc3813, you may be overthinking things a bit when you analyzed answer D on this one. We don't know whether the employees can currently afford housing closer than 30 minutes to work, and we don't know anything about their current commute. All we know is that right now, at their current income levels, they cannot afford to live within 30 minutes of the new location. Our author then concludes that they will end up with a 30+ minute commute. It's not about how their commute will change, but only about what it will be like when they move. Not necessary to hold this up to causal scrutiny, I think.
Hi Adam,

In your explanation above, you say "we don't know whether the employees can currently afford housing closer than 30 minutes to work, and we don't know anything about their current commute," but the stimulus reads:

"So once the company has moved, most Acme employees will have a commute of more than 30 minutes". Does this not imply that the 30 minute commute is a new thing? And one that will only come into play after the company relocates? It seems counter-intuitive for us to assume that most of the workers have a +30 minute commute to the Milltown office location, because if that were the case, there wouldn't be much need for the author's argument in the first place...?

I guess I'm most confused by the fact that the commute seems to be more central to the argument than the ability to afford housing - perhaps this is a trick employed by the LSAT writers, as "30 minutes" stands out given the numerical values, and this appears twice in the stimulus, whereas "cannot afford housing" is casually dropped in the middle, without much emphasis or importance.

How should I be looking at the argument differently in order to see what you are seeing? Thanks!
 Adam Tyson
PowerScore Staff
  • PowerScore Staff
  • Posts: 5153
  • Joined: Apr 14, 2011
|
#68606
I do think it's too much of an assumption to say that the employee commutes are currently not more than 30 minutes, ShannonOh22. While it's possible, and makes sense that the author wouldn't have mentioned it this way if that were not the case, it's not necessary. I would read that "once" as being no different than if the author had said "after," and telling us what will be true after something happens doesn't necessarily indicate what was true before it happened.

Also, try the negation technique on answer D. What if currently, most of the employees do NOT have a commute under 30 minutes? Maybe they all drive about 40 minutes to get to work? What impact does that have on the claim that after the move they will have that long of a commute? Looks like none to me - it could still be true that they will be unable to afford to live closer and will therefore live at least that far away. Since the negation has no impact on the argument, that answer cannot be a required assumption of the argument. It might be true, but it doesn't have to be true in order for the argument to stand, and that's the standard for evaluating any assumption answer.
 ShannonOh22
  • Posts: 70
  • Joined: Aug 15, 2019
|
#70914
Right you are, and I thank you for your explanation Adam. That was helpful - I see through the negation technique that my answer choice selection doesn't really do much of anything at all. :) Your response is greatly appreciated!

Get the most out of your LSAT Prep Plus subscription.

Analyze and track your performance with our Testing and Analytics Package.